Saltar al contenido principal
LibreTexts Español

2.1: Funciones y límites

  • Page ID
    110490
  • \( \newcommand{\vecs}[1]{\overset { \scriptstyle \rightharpoonup} {\mathbf{#1}} } \) \( \newcommand{\vecd}[1]{\overset{-\!-\!\rightharpoonup}{\vphantom{a}\smash {#1}}} \)\(\newcommand{\id}{\mathrm{id}}\) \( \newcommand{\Span}{\mathrm{span}}\) \( \newcommand{\kernel}{\mathrm{null}\,}\) \( \newcommand{\range}{\mathrm{range}\,}\) \( \newcommand{\RealPart}{\mathrm{Re}}\) \( \newcommand{\ImaginaryPart}{\mathrm{Im}}\) \( \newcommand{\Argument}{\mathrm{Arg}}\) \( \newcommand{\norm}[1]{\| #1 \|}\) \( \newcommand{\inner}[2]{\langle #1, #2 \rangle}\) \( \newcommand{\Span}{\mathrm{span}}\) \(\newcommand{\id}{\mathrm{id}}\) \( \newcommand{\Span}{\mathrm{span}}\) \( \newcommand{\kernel}{\mathrm{null}\,}\) \( \newcommand{\range}{\mathrm{range}\,}\) \( \newcommand{\RealPart}{\mathrm{Re}}\) \( \newcommand{\ImaginaryPart}{\mathrm{Im}}\) \( \newcommand{\Argument}{\mathrm{Arg}}\) \( \newcommand{\norm}[1]{\| #1 \|}\) \( \newcommand{\inner}[2]{\langle #1, #2 \rangle}\) \( \newcommand{\Span}{\mathrm{span}}\)\(\newcommand{\AA}{\unicode[.8,0]{x212B}}\)

    En esta sección se estudian los límites de las funciones de valor real de una variable real. Estudiaste límites en el cálculo. Sin embargo, veremos más cuidadosamente la definición de límite y probaremos teoremas que generalmente no se prueban en el cálculo.

    Una regla\(f\) que asigna a cada miembro de un conjunto no vacío\(D\) un miembro único de un conjunto\(Y\) es un {}. Escribimos la relación entre un miembro\(x\) de\(D\) y el miembro\(y\) de\(Y\) que se\(f\) asigna\(x\) como

    \[ y=f(x). \]

    El conjunto\(D\) es el {} de\(f\), denotado por\(D_f\). Los miembros de\(Y\) son los posibles {} de\(f\). Si\(y_0\in Y\) y hay una\(x_0\) en\(D\) tal que\(f(x_0)=y_0\) entonces decimos que\(f\) {} o {} el valor\(y_0\). El conjunto de valores obtenidos por\(f\) es el {} de\(f\). A {} es una función cuyo dominio y rango son ambos subconjuntos de los reales. Si bien en esta sección sólo nos preocupan las funciones de valor real de una variable real, nuestras definiciones no se limitan a esta situación. En secciones posteriores consideraremos situaciones donde el rango o dominio, o ambos, son subconjuntos de espacios vectoriales.

    Ejemplo\(\PageIndex{1}\)

    Ejemplo 2.1.1 Las funciones f, g y h definidas en .1; 1/ por f .x/ D x 2; g.x/ D sin x; y h.x/ D e x tienen rangos œ0; 1/, œ1; 1, y .0; 1/, respectivamente. Solución

    Agrega texto aquí.

    Ejemplo\(\PageIndex{2}\)

    Ejemplo 2.1.2 La ecuación œf .x/2 D x (2.1.1) no define una función excepto en el conjunto singleton f0g. Si x < 0, no real number satisfies (2.1.1), while if x > 0, dos números reales satisfacen (2.1.1). Sin embargo, las condiciones œf .x/2 D x y f .x/ 0 definen una función f en Df D œ0; 1/ con valores f .x/ D p x. Del mismo modo, las condiciones œg.x/2 D x y g.x/ 0 definen una función g en Dg D œ0; 1/ con valores g.x/ D p x. Los rangos de f y g son œ0; 1/ y .1; 0, respectivamente. Solución

    Agrega texto aquí.

    Es importante entender que la definición de una función incluye la especificación de su dominio y que existe una diferencia entre f, el nombre de la función, y f .x/, el valor de f en x Sin embargo, la estricta observancia de estos puntos conduce a una verbosidad molesta, como “la función f con dominio .1; 1/ y valores f .x/ D x.” Esto lo evitaremos de dos maneras: (1) acordando que si se introduce una función f sin definir explícitamente Df, entonces se entenderá que Df consiste en todos los puntos x para los que tiene sentido la regla que define f .x/, y (2) teniendo en cuenta la distinción entre f y f .x/, pero no enfatizándola cuando sería una molestia hacerlo. Por ejemplo, vamos a escribir “considerar la función f .x/ D p 1 x 2”, en lugar de “considerar la función f definida en œ1; 1 por f .x/ D p 1 x 2”, o “considerar la función g.x/ D 1= sin x”, en lugar de “considerar la función g definida para x ¤ k (k D entero) por g.x/ D 1= sin x.” También escribiremos f D c (constante) para denotar la función f definida por f .x/ D c para todas x. Nuestra definición de función es algo intuitiva, pero adecuada para nuestros propósitos. Además, es la forma de trabajo de la definición, aunque la idea se introduzca de manera más rigurosa para empezar. Para una definición más precisa, primero definimos el producto cartesiano X Y de dos conjuntos no vacíos X e Y para que sea el conjunto de todos los pares ordenados .x; y/ tal que x 2 X e y 2 Y; así, X Y D ˚ .x; y/ x 2 X; y

    Un subconjunto no vacío\(f\) de\(X\times Y\) es una función si no\(x\) in\(X\) ocurre más de una vez como primer miembro entre los elementos de\(f\). Dicho de otra manera, si\((x,y)\) y\((x,y_1)\) están adentro\(f\), entonces\(y=y_1\). El conjunto de\(x\)'s que ocurren como primeros miembros de\(f\) es el de\(f\). Si\(x\) está en el dominio de\(f\), entonces lo único\(y\) en\(Y\) tal que\((x,y)\in f\) es el {}, y escribimos\(y=f(x)\). El conjunto de todos esos valores, un subconjunto de\(Y\), es el rango de f\(f\).

    Operaciones Aritméticas en Funciones

    La esencia del concepto de límite para funciones de valor real de una variable real es esta: Si\(L\) es un número real, entonces\(\lim_{x\to x_0}f(x)=L\) significa que el valor se\(f(x)\) puede hacer lo más cerca que deseemos tomando\(x\) suficientemente cerca a\(x_0\).\(L\) Esto se hace preciso en la siguiente definición.

    Destacamos que Definición~ no implica\(f(x_0)\), ni siquiera requiere que se defina, ya que excluye el caso donde\(x=x_0\).

    El siguiente teorema dice que una función no puede tener más de un límite en un punto.

    Supongamos que sostiene y deja\(\epsilon>0\). Desde Definición~, hay números positivos\(\delta_1\) y\(\delta_2\) tales que\[ |f(x)-L_i|<\epsilon\mbox{\quad if \quad} 0<|x-x_0|<\delta_i, \quad i=1,2. \] Si\(\delta=\min(\delta_1,\delta_2)\), entonces Ahora\[\begin{eqnarray*} |L_1-L_2|\ar= |L_1-f(x)+f(x)-L_2|\\ \ar \le|L_1-f(x)|+|f(x)-L_2|<2\epsilon \mbox{\quad if \quad} 0<|x-x_0|<\delta. \end{eqnarray*}\] hemos establecido una desigualdad de la que no depende\(x\); es decir,\[ |L_1-L_2|<2\epsilon. \] ya que esto sostiene para cualquier positivo\(\epsilon\),\(L_1=L_2\).

    La definición~ no se cambia reemplazando con\[\begin{equation}\label{eq:2.1.8} |f(x)-L|<K\epsilon, \end{equation}\] donde\(K\) es una constante positiva, porque si cualquiera de o se puede hacer que se sostenga para cualquiera\(\epsilon>0\) haciendo\(|x-x_0|\) lo suficientemente pequeño y positivo, entonces también lo puede hacer el otro (Ejercicio~). Esto puede parecer un punto menor, pero a menudo es conveniente trabajar con él en lugar de, como veremos en la prueba del siguiente teorema.

    De y Definición~\(\epsilon>0\), si, hay\(\delta_1>0\) tal que\[\begin{equation}\label{eq:2.1.14} |f(x)-L_1|<\epsilon \end{equation}\] si\(0<|x-x_0|<\delta_1\), y\(\delta_2>0\) tal que\[\begin{equation}\label{eq:2.1.15} |g(x)-L_2|<\epsilon \end{equation}\] si\(0<|x-x_0|<\delta_2\). Supongamos\[\begin{equation}\label{eq:2.1.16} 0<|x-x_0|<\delta=\min (\delta_1,\delta_2), \end{equation}\] que para eso y ambos aguanten. Entonces\[\begin{eqnarray*} |(f\pm g)(x)-(L_1\pm L_2)|\ar= |(f(x)-L_1)\pm (g(x)-L_2)|\\ \ar \le|f(x)-L_1|+|g(x)-L_2|<2\epsilon, \end{eqnarray*}\] lo que prueba y.

    Para probar, asumimos y escribimos\[\begin{eqnarray*} |(fg)(x)-L_1L_2|\ar= |f(x)g(x)-L_1L_2|\\[.5\jot] \ar= |f(x)(g(x)-L_2)+L_2(f(x)-L_1)|\\[.5\jot] \ar \le|f(x)||g(x)-L_2|+|L_2||f(x)-L_1|\\[.5\jot] \ar \le(|f(x)|+|L_2|)\epsilon\mbox{\quad (from \eqref{eq:2.1.14} and \eqref{eq:2.1.15})}\\[.5\jot] \ar \le(|f(x)-L_1|+|L_1|+|L_2|)\epsilon\\[.5\jot] \ar \le(\epsilon+|L_1|+|L_2|)\epsilon\mbox{\quad from \eqref{eq:2.1.14}}\\[.5\jot] \ar \le (1+|L_1|+|L_2|)\epsilon \end{eqnarray*}\] si\(\epsilon<1\) y\(x\) satisface. Esto prueba.

    Para probar, primero observamos que si\(L_2\ne0\), hay\(\delta_3>0\) tal que\[ |g(x)-L_2|<\frac{|L_2|}{2}, \] así que\[\begin{equation} \label{eq:2.1.17} |g(x)|>\frac{|L_2|}{2} \end{equation}\] si\[ 0<|x-x_0|<\delta_3. \] Para ver esto, dejar\(L=L_2\) y\(\epsilon=|L_2|/2\) entrar. Ahora supongamos\[ 0<|x-x_0|<\min (\delta_1,\delta_2,\delta_3), \] eso para que,, y todos aguanten. Entonces

    Esto prueba.

    Las aplicaciones sucesivas de las diversas partes del Teoremo~ nos permiten encontrar límites sin los\(\epsilon\)\(\delta\) argumentos requeridos por Definición~.

    -.2em La función\[ f(x)=2x\,\sin\sqrt{x} \]

    satisface la desigualdad\[ |f(x)|<\epsilon \] si\(0<x<\delta=\epsilon/2\). Sin embargo, esto no quiere decir que\(\lim_{x \to 0} f(x)=0\), ya que no\(f\) se define para negativo\(x\), como debe ser para satisfacer las condiciones de Definición~ con\(x_0=0\) y\(L=0\). La función se\[ g(x)=x+\frac{|x|}{ x},\quad x\ne0, \] puede reescribir ya que\[ g(x)=\left\{\casespace\begin{array}{ll} x+1,&x>0,\\ x-1,&x<0;\end{array}\right. \] por lo tanto, cada intervalo abierto que contiene\(x_0=0\) también contiene puntos\(x_1\) y\(x_2\) tal que\(|g(x_1)-g(x_2)|\) está lo más cerca que\(2\) nos plazca. Por lo tanto,\(\lim_{x\to x_0} g(x)\) no existe (Ejercicio~).

    Aunque\(f(x)\) y\(g(x)\) no se acercan a los límites como se\(x\) acerca a cero, cada uno de ellos exhibe un tipo definido de comportamiento limitante para pequeños valores positivos de\(x\), como lo hace\(g(x)\) para pequeños valores negativos de\(x\). El tipo de comportamiento que tenemos en mente se define precisamente de la siguiente manera.

    6pt

    12pt

    Figura~ muestra la gráfica de una función que tiene distintos límites a la izquierda y a la derecha en un punto\(x_0\).

    Los límites izquierdo y derecho también se llaman {}. A menudo simplificaremos la notación escribiendo\[ \lim_{x\to x_0-} f(x)=f(x_0-)\mbox{\quad and \quad}\lim_{x\to x_0+} f(x)=f(x_0+). \]

    El siguiente teorema establece la conexión entre límites y límites unilaterales. Te dejamos la prueba (Ejercicio~).

    Con sólo modificaciones menores de sus pruebas (sustituyendo la desigualdad\(0<|x-x_0|<\delta\) por\(x_0-\delta<x<x_0\) o\(x_0<x<x_0+ \delta\)), se puede demostrar que las aseveraciones de Teoremas ~ y siguen siendo válidas si $\ lim_ {x\ to x_0} $” se sustituye por\(\lim_{x\to x_0-}\) ''o ``\(\lim_{x\to x_0+}\)” a lo largo de (Ejercicio~).

    Los límites y los límites unilaterales tienen que ver con el comportamiento de una función\(f\) cerca de un punto límite de\(D_f\). Es igualmente razonable estudiar\(f\) para grandes valores positivos de\(x\) si no\(D_f\) está delimitado por encima o para grandes valores negativos de\(x\) si no\(D_f\) está delimitado por debajo.

    Figura~ proporciona una ilustración de la situación descrita en Definición~.

    6pt

    12pt

    Te dejamos a ti definir la sentencia ``\(\lim_{x\to-\infty} f(x)=L\)” (Ejercicio~) y mostrar que Teoremas ~ y que permanezcan vigentes si\(x_0\) se sustituye a lo largo de todo por\(\infty\) o\(-\infty\) (Ejercicio~).

    A veces vamos a denotar\(\lim_{x\to\infty}f(x)\) y\(\lim_{x \to-\infty}f(x)\) por\(f(\infty)\) y\(f(-\infty)\), respectivamente.

    -.3em Las funciones\[ f(x)=\frac{1}{ x},\quad g(x)=\frac{1}{ x^2},\quad p(x)=\sin\frac{1}{ x}, \] y\[ q(x)=\frac{1}{ x^2}\sin \frac{1}{ x} \] no tienen límites, o incluso límites unilaterales, en\(x_0=0\). No logran tener límites de diferentes maneras:

    El tipo de comportamiento exhibido por\(f\) y\(g\) cerca\(x_0=0\) es lo suficientemente común y sencillo como para llevarnos a definir {}.

    A lo largo de este libro, “\(\lim_{x\to x_0} f(x)\)existe” significará que

    \[\lim_{x\to x_0} f(x)=L,\quad where L is {\it finite\/}.}\]

    Para dejar abierta la posibilidad de que\(L=\pm \infty\), diremos que\[ \lim_{x\to x_0} f(x)\quad\mbox{\it exists in the extended reals.\/} \] Esta convención también se aplica a los límites unilaterales y a los límites como\(x\) enfoques\(\pm\infty\).

    Mencionamos anteriormente que Teoremas ~ y siguen siendo válidos si $\ lim_ {x\ to x_0} $” se sustituye por\(\lim_{x\to x_0-}\) ''o ``”\(\lim_{x\to x_0+}\). También son válidos con\(x_0\) reemplazados por\(\pm\infty\). Además, las contrapartes de,, y en todas estas versiones del Teorem~ siguen siendo válidas si alguna o ambas de\(L_1\) y\(L_2\) son infinitas, siempre que sus lados derechos no sean indeterminados (Ejercicios~ y). Ecuación y sus\(L_1/L_2\) contrapartes siguen siendo válidas si no es indeterminada y\(L_2\ne0\) (Ejercicio~).

    -.3em Una función\(f\) es {} en un intervalo\(I\) si\[\begin{equation}\label{eq:2.1.18} f(x_1)\le f(x_2)\quad\mbox{whenever $x_1$ and $x_2$ are in $I$ and $x_1 <x_2$}, \end{equation}\] o {} on\(I\) si\[\begin{equation}\label{eq:2.1.19} f(x_1)\ge f(x_2)\quad\mbox{whenever $x_1$ and $x_2$ are in $I$ and $x_1 <x_2$}. \end{equation}\] En cualquier caso,\(f\) está activado\(I\). Si\(\le\) puede ser reemplazado por\(<\) in,\(f\) es {} encendido\(I\). Si\(\ge\) puede ser reemplazado por\(>\) in,\(f\) es {} encendido\(I\). En cualquiera de estos dos casos,\(f\) está {} encendido\(I\).

    6pt

    12pt

    En la prueba del siguiente teorema, asumimos que usted ha formulado las definiciones que se piden en Ejercicio~.

    Primero lo demostramos\(f(a+)=\alpha\). Si

    \(M>\alpha\), hay un\(x_0\) en\((a,b)\) tal que\(f(x_0)<M\). Ya que\(f\) es no decreciente,\(f(x)<M\) si\(a<x<x_0\). Por lo tanto, si\(\alpha=-\infty\), entonces\(f(a+)=-\infty\). Si\(\alpha>-\infty\), vamos\(M=\alpha+\epsilon\), donde\(\epsilon>0\). Entonces\(\alpha\le f(x)<\alpha+\epsilon\), entonces\[\begin{equation} \label{eq:2.1.20} |f(x)-\alpha|<\epsilon\mbox{\quad if \quad} a<x<x_0. \end{equation}\] si\(a=-\infty\), esto implica eso\(f(-\infty)=\alpha\). Si\(a>-\infty\), vamos\(\delta=x_0-a\). Entonces es equivalente a\[ |f(x)-\alpha|<\epsilon\mbox{\quad if \quad} a<x<a+\delta, \] lo que implica eso\(f(a+)=\alpha\).

    Ahora lo demostramos\(f(b-)=\beta\). Si\(M<\beta\), hay una\(x_0\) en\((a,b)\) tal que\(f(x_0)>M\). Ya que\(f\) es no decreciente,\(f(x)>M\) si\(x_0<x<b\). Por lo tanto, si\(\beta=\infty\), entonces\(f(b-)=\infty\). Si\(\beta<\infty\), vamos\(M=\beta-\epsilon\), donde\(\epsilon>0\). Entonces\(\beta-\epsilon< f(x)\le\beta\), entonces\[\begin{equation} \label{eq:2.1.21} |f(x)-\beta|<\epsilon\mbox{\quad if \quad} x_0<x<b. \end{equation}\] si\(b=\infty\), esto implica eso\(f(\infty)=\beta\). Si\(b<\infty\), vamos\(\delta=b-x_0\). Entonces es equivalente a\[ |f(x)-\beta|<\epsilon\mbox{\quad if \quad} b-\delta<x<b, \] lo que implica eso\(f(b-)=\beta\).

    La prueba es similar a la prueba de

    (Ejercicio~).

    Supongamos que\(f\) es no decreciente. Aplicando

    a\(f\) on\((a,x_0)\) y\((x_0,b)\) por separado muestra que\[ f(x_0-)=\sup_{a<x<x_0}f(x)\mbox{\quad and \quad} f(x_0+)=\inf_{x_0<x<b}f(x). \]

    Sin embargo\(x_1<x_0<x_2\), si,\[ f(x_1)\le f(x_0)\le f(x_2); \] entonces,\[ f(x_0-)\le f(x_0)\le f(x_0+). \]

    Te dejamos el caso donde no\(f\) es creciente para ti (Ejercicio~).

    Ahora introducimos algunos conceptos relacionados con los límites. Dejamos el estudio de estos conceptos principalmente a los ejercicios.

    Decimos que\(f\) es {} en un set\(S\) si hay una constante\(M<\infty\) tal que\(|f(x)|\le M\) para todos\(x\) en\(S\).

    Ya que\(S_f(x_1;x_0)\) es lo supremo de\(\set{f(t)}{x_1<t<x_0}\), hay una\(\overline x\) en\([x_1,x_0)\) tal que\[ f(\overline x)>S_f(x_1;x_0)-\epsilon/2. \] Esto e implica eso\(f(\overline x)>\beta-\epsilon\). Ya que\(\overline x\) está en\([a_1,x_0)\), esto prueba

    .

    Ahora demostramos que no puede haber más de un número real con propiedades y. Supongamos que\(\beta_1<\beta_2\) y\(\beta_2\) tiene propiedad; así, si\(\epsilon>0\) y\(a_1\) está en\([a,x_0)\), hay una\(\overline x\) en\([a_1,x_0)\) tal que\(f(\overline x)>\beta_2-\epsilon\). Dejando\(\epsilon=\beta_2-\beta_1\), vemos que hay un\(\overline x\) en\([a_1,b)\) tal que\[ f(\overline x)>\beta_2-(\beta_2-\beta_1)=\beta_1, \] así\(\beta_1\) no puede tener propiedad. Por lo tanto, no puede haber más de un número real que satisfaga tanto y

    .

    La prueba del siguiente teorema es similar a esta (Ejercicio~).

    En esta sección se estudian las funciones continuas de una variable real. Demostraremos algunos teoremas importantes sobre funciones continuas que, aunque intuitivamente plausibles, están más allá del alcance del curso de cálculo elemental. Son accesibles ahora por nuestra mejor comprensión del sistema de números reales, especialmente de aquellas propiedades que se derivan del axioma de integridad.

    Las definiciones de\[ f(x_0-)=\lim_{x\to x_0-}f(x),\quad f(x_0+)=\lim_{x\to x_0+}f(x),\mbox{\quad and \quad} \lim_{x\to x_0} f(x) \] no implican\(f(x_0)\) ni requieren que se defina. Sin embargo,\(f(x_0)\) es importante el caso donde se define e igual a una o más de estas cantidades.

    El siguiente teorema proporciona un método para determinar si estas definiciones están satisfechas. El comprobante, que te dejamos (Ejercicio~), descansa en Definiciones~,, y.

    De Definición~ y Teorem~,\(f\) es

    continuo en\(x_0\) si y solo si\[ f(x_0-)=f(x_0+)=f(x_0) \] o, equivalentemente, si y solo si es continuo desde la derecha y la izquierda en\(x_0\) (Ejercicio~).

    6pt

    12pt

    La función\(f\) definida en Ejemplo~ (ver también Figura ~) es continua en\([0,1)\) y\([1,2]\), pero no en ningún intervalo abierto que contenga 1. La discontinuidad de\(f\) allí es del tipo más simple, descrito en la siguiente definición.

    6pt

    12pt

    El motivo del adjetivo ``salto” se puede ver en Figuras~ y, donde las gráficas exhiben un salto definido en cada punto de discontinuidad. El siguiente ejemplo muestra que no todas las discontinuidades son de este tipo.

    Teoremas ~ e implican el siguiente teorema (Ejercicio~).

    Dejar\(f\) definirse en una vecindad eliminada de\(x_0\) y discontinua (tal vez incluso indefinida) en\(x_0\). Decimos que\(f\) tiene un at\(x_0\) si\(\lim_{x\to x_0}f(x)\) existe. En este caso, la función\[ g(x)=\left\{\casespace\begin{array}{ll} f(x)&\mbox{ if } x\in D_f\mbox{ and } x\ne x_0,\\[2\jot] \dst{\lim_{x\to x_0}} f(x)&\mbox{ if } x=x_0,\end{array}\right. \] es continua en\(x_0\).

    Hemos visto que la investigación de límites y continuidad puede simplificarse considerando una función dada como resultado de la suma, resta, multiplicación y división de funciones más simples. Otra operación útil en este sentido es {} de funciones; es decir, la sustitución de una función por otra.

    El siguiente teorema dice que la composición de las funciones continuas es continua.

    Supongamos que\(\epsilon>0\). Ya que\(g(x_0)\) es un punto interior de\(D_f\) y\(f\) es continuo en\(g(x_0)\), hay un\(\delta_1>0\) tal que\(f(t)\) se define y\[\begin{equation}\label{eq:2.2.4} |f(t)-f(g(x_0))|<\epsilon\mbox{\quad if \quad} |t-g(x_0)|< \delta_1. \end{equation}\] Dado que\(g\) es continuo en\(x_0\), hay un\(\delta>0\) tal que\(g(x)\) se define y\[\begin{equation}\label{eq:2.2.5} |g(x)-g(x_0)|<\delta_1\mbox{\quad if \quad}|x-x_0|<\delta. \end{equation}\] Ahora e implica que \[ |f(g(x))-f(g(x_0))|<\epsilon\mbox{\quad if \quad}|x-x_0|<\delta. \]Por lo tanto,\(f\circ g\) es continuo en\(x_0\).

    Consulte Ejercicio~ para obtener un resultado relacionado con los límites.

    Una función\(f\) es {} en un conjunto\(S\) si hay un número real\(m\) tal que\[ f(x)\ge m \mbox{ for all } x\in S. \] en este caso, el conjunto\[ V=\set{f(x)}{x\in S} \] tiene un infimum\(\alpha\), y escribimos\[ \alpha=\inf_{x\in S}f(x). \] Si hay un punto\(x_1\) en\(S\) tal que\(f(x_1)=\alpha\), decimos que\(\alpha\) es el {}, y escribimos \[ \alpha=\min_{x\in S}f(x). \]De igual manera,\(f\) es {} si hay un número real\(M\) tal que\(f(x)\le M\) para todos\(x\) en\(S\). En este caso,\(V\) tiene un supremum\(\beta\), y escribimos\[ \beta=\sup_{x\in S}f(x). \] Si hay un punto\(x_2\) en\(S\) tal que\(f(x_2)=\beta\), decimos que\(\beta\) es el {}, y escribimos\[ \beta=\max_{x\in S}f(x). \] Si\(f\) está acotado arriba y abajo en un set\(S\), decimos que\(f\) es {} on \(S\).

    Figure~ ilustra el significado geométrico de estas definiciones para una función\(f\) delimitada en un intervalo\(S=[a,b]\). La gráfica de\(f\) yace en la franja delimitada por las líneas\(y=M\) y\(y=m\), donde\(M\) está cualquier límite superior y\(m\) es cualquier límite inferior para\(f\) on\([a,b]\). La franja más estrecha que contiene la gráfica es la delimitada arriba\(y=\beta= \sup_{a\le x\le b}f(x)\) y abajo por\(y=\alpha=\inf_{a\le x\le b}f(x)\).

    6pt

    12pt

    6pt

    12pt

    Supongamos que\(t\in [a,b]\). Ya que\(f\) es continuo en\(t\), hay un intervalo abierto\(I_t\) que contiene\(t\) tal que\[\begin{equation}\label{eq:2.2.7} |f(x)-f(t)|<1 \mbox{\quad if \quad}\ x\in I_t\cap [a,b]. \end{equation}\] (Para ver esto, establecer\(\epsilon=1\) en, Teorem~.) La colección\({\mathcal H}=\set{I_t}{a\le t\le b}\) es una cubierta abierta de\([a,b]\). Dado que\([a,b]\) es compacto, el teorema de Heine—Borel implica que hay finitamente muchos puntos\(t_1\)\(t_2\),,,\(t_n\) tales que los intervalos\(I_{t_1}\),\(I_{t_2}\),,\(I_{t_n}\) cubren\([a,b]\). Según con\(t=t_i\),\[ |f(x)-f(t_i)|<1\mbox{\quad if \quad}\ x\in I_{t_i}\cap [a,b]. \] Por lo tanto,\[\begin{equation}\label{eq:2.2.8} \begin{array}{rcl} |f(x)|\ar =|(f(x)-f(t_i))+f(t_i)|\le|f(x)-f(t_i)|+|f(t_i)|\\[2\jot] \ar\le 1+|f(t_i)|\mbox{\quad if \quad}\ x\in I_{t_i}\cap[a,b]. \end{array} \end{equation}\] Let\[ M=1+\max_{1\le i\le n}|f(t_i)|. \]\([a,b]\subset\bigcup^n_{i=1}\left(I_{t_i}\cap [a,b]\right)\) Since, implica que\(|f(x)|\le M\) si\(x\in [a,b]\).

    Esta prueba ilustra la utilidad del teorema de Heine—Borel, que nos permite elegir\(M\) como el mayor de un {} conjunto de números.

    Teoremo~ y la integridad de los reales implican que

    si\(f\) es continuo en un intervalo cerrado finito\([a,b]\), entonces\(f\) tiene un infimum y un supremum encendido\([a,b]\). El siguiente teorema muestra que\(f\) realmente asume estos valores en algunos puntos de\([a,b]\).

    Mostramos que\(x_1\) existe y te lo dejamos a ti para que demuestres que\(x_2\) existe (Ejercicio~).

    Supongamos que no hay\(x_1\) en\([a,b]\) tal que\(f(x_1)=\alpha\). Entonces\(f(x)>\alpha\) para todos\(x\in[a,b]\). Demostraremos que esto lleva a una contradicción.

    Supongamos que\(t\in[a,b]\). Entonces\(f(t)>\alpha\), entonces\[ f(t)>\frac{f(t)+\alpha}{2}>\alpha. \]

    Dado que\(f\) es continuo en\(t\), hay un intervalo abierto\(I_t\) sobre\(t\) tal que\[\begin{equation}\label{eq:2.2.9} f(x)>\frac{f(t)+\alpha}{2}\mbox{\quad if \quad} x\in I_t\cap [a,b] \end{equation}\] (Ejercicio~). La colección\({\mathcal H}=\set{I_t}{a\le t\le b}\) es una cubierta abierta de\([a,b]\). Dado que\([a,b]\) es compacto, el teorema de Heine—Borel implica que hay finitamente muchos puntos\(t_1\)\(t_2\),,,\(t_n\) tales que los intervalos\(I_{t_1}\),\(I_{t_2}\),,\(I_{t_n}\) cubren\([a,b]\). Definir\[ \alpha_1=\min_{1\le i\le n}\frac{f(t_i)+\alpha}{2}. \] Entonces, ya que\([a,b]\subset\bigcup^n_{i=1} (I_{t_i}\cap [a,b])\), implica que\[ f(t)>\alpha_1,\quad a\le t\le b. \] Pero\(\alpha_1>\alpha\), así esto contradice la definición de\(\alpha\). Por lo tanto,\(f(x_1)=\alpha\) para algunos\(x_1\) en\([a,b]\).

    El siguiente teorema muestra que si\(f\) es continuo en un intervalo cerrado finito\([a,b]\), entonces\(f\) asume cada valor entre\(f(a)\) y\(f(b)\) como\(x\) varía de\(a\) a\(b\) (Figura, página).

    6pt

    12pt

    Supongamos que\(f(a)<\mu<f(b)\). El conjunto\[ S=\set{x}{a\le x\le b \mbox{\quad and }\ f(x)\le\mu} \] es acotado y no vacío. Vamos\(c=\sup S\). Eso lo demostraremos\(f(c)=\mu\). Si\(f(c)>\mu\), entonces\(c>a\) y, ya que\(f\) es continuo en\(c\), hay\(\epsilon>0\) tal que\(f(x)>\mu\) si\(c-\epsilon<x\le c\) (Ejercicio~). Por lo tanto,\(c-\epsilon\) es un límite superior para\(S\), que contradice la definición de\(c\) como supremo de\(S\). Si\(f(c) <\mu\), entonces\(c<b\) y hay\(\epsilon>0\) tal que\(f(x)<\mu\) para\(c\le x<c+\epsilon\), así no\(c\) es un límite superior para\(S\). Esto también es una contradicción. Por lo tanto,\(f(c)=\mu\).

    El comprobante para el caso donde se\(f(b)<\mu<f(a)\) pueda obtener aplicando este resultado a\(-f\).

    Teorem~ y Definición~ implican que un

    función\(f\) es continua en un subconjunto\(S\) de su dominio si para cada uno\(\epsilon>0\) y cada uno\(x_0\) en\(S\), hay un\(\delta>0\), {}, tal que\[ |f(x)-f(x_0)|<\epsilon\mbox{\quad if \quad} |x-x_0|<\delta \mbox{\quad and \quad} x\in D_f. \]

    La siguiente definición introduce otro tipo de continuidad en un set\(S\).

    Destacamos que en esta definición\(\delta\) depende sólo de\(\epsilon\) y\(S\) y no de la elección particular de\(x\) y\(x'\), siempre que ambos estén en\(S\).

    A menudo un concepto se aclara considerando su negación: una función\(f\) es {} uniformemente continua en\(S\) si hay\(\epsilon_0>0\) tal que si\(\delta\) hay algún número positivo, hay puntos\(x\) y\(x'\) en\(S\) tal que\[ |x-x'|<\delta\mbox{\quad but\quad} |f(x)-f(x')|\ge\epsilon_0. \]

    Ejemplos~ y muestran que una función puede ser continua pero no uniformemente continua en un intervalo. El siguiente teorema muestra que esto no puede suceder si el intervalo es cerrado y acotado, y por lo tanto compacto.

    Supongamos que\(\epsilon>0\). Ya que\(f\) es continuo encendido\([a,b]\), para cada uno\(t\) en\([a,b]\) hay un número positivo\(\delta_{t}\) tal que\[\begin{equation}\label{eq:2.2.10} |f(x)-f(t)|<\frac{\epsilon}{2} \mbox{\quad if \quad} |x-t|<2\delta_{t} \mbox{\quad and \quad} x\in[a,b]. \end{equation}\] Si\(I_{t}=(t-\delta_{t },t+\delta_{t})\), la colección\[ {\mathcal H}=\set{I_{t}}{t\in [a,b]} \] es una cubierta abierta de\([a,b]\). Dado que\([a,b]\) es compacto, el teorema de Heine—Borel implica que hay finitamente muchos puntos\(t_1\)\(t_2\),,,\(t_n\) en\([a,b]\) tal que\(I_{t_1}\),\(I_{t_2}\),,\(I_{t_n}\) cubren\([a,b]\). Ahora\[\begin{equation}\label{eq:2.2.11} \delta=\min\{\delta_{t_1},\delta_{t_2}, \dots,\delta_{t_n}\}. \end{equation}\] definimos Vamos a mostrar que si\[\begin{equation} \label{eq:2.2.12} |x-x'|<\delta \mbox{\quad and \quad}x,x'\in [a,b], \end{equation}\] entonces\(|f(x)-f(x')|<\epsilon\).

    Desde el triángulo la desigualdad,\[\begin{equation} \label{eq:2.2.13} \begin{array}{rcl} |f(x)-f(x')|\ar = |\left(f(x)-f(t_r)\right)+\left(f(t_r)-f(x')\right)|\\ \ar\le |f(x)-f(t_r)|+|f(t_r)-f(x')|. \end{array} \end{equation}\] ya que\(I_{t_1}\)\(I_{t_2}\),,,\(I_{t_n}\) cubrir\([a,b]\),\(x\) debe estar en uno de estos intervalos. Supongamos que\(x\in I_{t_r}\); es decir,\[\begin{equation} \label{eq:2.2.14} |x-t_r|<\delta_{t_r}. \end{equation}\] De con\(t=t_r\),\[\begin{equation} \label{eq:2.2.15} |f(x)-f(t_r)|<\frac{\epsilon}{2}. \end{equation}\] De, y el triángulo incalidad,\[ |x'-t_r|=|(x'-x)+(x-t_r)|\le |x'-x|+|x-t_r|<\delta+\delta_{t_r}\le2\delta_{t_r}. \] Por lo tanto, con\(t=t_r\) y\(x\) reemplazado por\(x'\) implica que\[ |f(x')-f(t_r)|<\frac{\epsilon}{2}. \] Esto,, e implica que\(|f(x)-f(x')|<\epsilon\).

    Esta prueba vuelve a mostrar la utilidad del teorema de Heine—Borel, que nos permitió definir\(\delta\) in como el más pequeño de un {} conjunto de números positivos, por lo que\(\delta\) seguramente será positivo. (Un conjunto infinito de números positivos puede no tener un miembro positivo más pequeño; por ejemplo, considere el intervalo abierto)\((0,1)\).

    Aplicado a Ejemplo~, Corolary~ implica que la función\(g(x)=\cos1/x\) es uniformemente continua en\([\rho,1]\) si\(0<\rho<1\).

    Teorem~ implica que si\(f\) es monótono en un intervalo\(I\), entonces\(f\) es continuo o tiene una discontinuidad de salto\(x_0\) en cada una de ellas\(I\). Esto y Teorem~ proporcionan la clave para la prueba del siguiente teorema.

    Asumimos que\(f\) es no decreciente, y dejamos el caso donde no\(f\) está aumentando para ti (Ejercicio). Teorema

    implica que el conjunto\(\widetilde R_f=\set{f(x)}{x\in(a,b)}\) es un subconjunto del intervalo abierto\((f(a+),f(b-))\). Por lo tanto,\[\begin{equation} \label{eq:2.2.16} R_f=\{f(a)\}\cup\widetilde R_f\cup\{f(b)\}\subset\{f(a)\}\cup(f(a+),f(b-))\cup\{f(b)\}. \end{equation}\] Ahora supongamos que\(f\) es continuo en\([a,b]\). Entonces\(f(a)=f(a+)\),\(f(b-)=f(b)\), así implica eso\(R_f\subset[f(a),f(b)]\). Si\(f(a)<\mu<f(b)\), entonces Teorem~ implica que\(\mu=f(x)\) para algunos\(x\) en\((a,b)\). De ahí,\(R_f=[f(a),f(b)]\).

    Para lo contrario, supongamos que\(R_f=[f(a),f(b)]\). Desde\(f(a)\le f(a+)\) y\(f(b-)\le f(b)\), implica que\(f(a)=f(a+)\) y\(f(b-)=f(b)\). Sabemos por Teorem~

    que si\(f\) es no decreciente y\(a<x_0<b\), entonces\[ f(x_0-)\le f(x_0)\le f(x_0+). \] Si alguna de estas desigualdades es estricta,\(R_f\) no puede ser un intervalo. Dado que esto contradice nuestra suposición,\(f(x_0-)=f(x_0)=f(x_0+)\). Por lo tanto,\(f\) es continuo en\(x_0\) (Ejercicio~). Ahora podemos concluir que\(f\) es continuo en\([a,b]\).

    Teorema ~ implica el siguiente teorema.

    Primero demostramos que hay una función\(g\) satisfactoria y. Ya que\(f\) es continuo, Teorem~ implica que para cada uno\(y_0\) en\([c,d]\) hay un\(x_0\) en\([a,b]\) tal que\[\begin{equation}\label{eq:2.2.19} f(x_0)=y_0, \end{equation}\]

    y, como\(f\) va en aumento, sólo hay uno de esos\(x_0\). Definir\[\begin{equation}\label{eq:2.2.20} g(y_0)=x_0. \end{equation}\] La definición de\(x_0\) se ilustra en Figura~: con\([c,d]\) dibujado en el\(y\) eje -eje, encontrar la intersección de la línea\(y=y_0\) con la curva\(y=f(x)\) y soltar una vertical desde la intersección hasta el\(x\) -eje para encontrar\(x_0\).

    6pt

    12pt

    Sustituir en rendimientos\[ f(g(y_0))=y_0, \] y sustituir por rendimientos\[ g(f(x_0))=x_0. \] Bajando los subíndices en estas dos ecuaciones rendimientos y.

    La singularidad de se\(g\) desprende de nuestra suposición que\(f\) va en aumento, y por lo tanto sólo un valor de\(x_0\) puede satisfacer para cada uno\(y_0\).

    Para ver eso\(g\) va en aumento, supongamos eso\(y_1<y_2\) y dejar\(x_1\) y\(x_2\) ser los puntos en\([a,b]\) tal que\(f(x_1)=y_1\) y\(f(x_2)=y_2\). Ya que\(f\) va en aumento,\(x_1<x_2\). Por lo tanto,\[ g(y_1)=x_1<x_2=g(y_2), \] así\(g\) es cada vez mayor. Ya que\(R_g=\set{g(y)}{y\in[c,d]}\) es el intervalo\([g(c),g(d)]=[a,b]\), Teorem~ con\(f\) y\([a,b]\) reemplazado por\(g\) e\([c,d]\) implica que\(g\) es continuo en\([c,d]\).

    La función\(g\) del Teorem~ es la {} de\(f\), denotada por\(f^{-1}\). Dado que y son simétricos en\(f\) y\(g\), también podemos considerar\(f\) como el inverso de\(g\), y denotarlo por\(g^{-1}\).

    En el cálculo se estudió la diferenciación, enfatizando reglas para calcular derivados. Aquí consideramos las propiedades teóricas de las funciones diferenciables. Al hacer esto, asumimos que sabes diferenciar funciones elementales como\(x^n\),\(e^x\), y\(\sin x\), y usaremos dichas funciones en ejemplos.

    Si\(f\) se define en un conjunto abierto\(S\), decimos que\(f\) es {}\(S\) si\(f\) es diferenciable en cada punto de\(S\). Si\(f\) es diferenciable en\(S\), entonces\(f'\) es una función en\(S\). Decimos que\(f\) es {} on\(S\) si\(f'\) es continuo encendido\(S\). Si\(f\) es diferenciable en un barrio de\(x_0\), es razonable preguntar si\(f'\) es diferenciable en\(x_0\). Si es así, denotamos la derivada de\(f'\) at\(x_0\) by\(f''(x_0)\). Este es el {}, y también se denota por\(f^{(2)}(x_0)\). Continuando inductivamente, si\(f^{(n-1)}\) se define en un vecindario de\(x_0\), entonces el\(n\) th {}, denotado por\(f^{(n)}(x_0)\), es la derivada de\(f^{(n-1)}\) at\(x_0\). Por conveniencia definimos el {} de\(f\) ser\(f\) él mismo; así\[ f^{(0)}=f. \]

    Asumimos que está familiarizado con las otras notaciones estándar para derivados; por ejemplo,\[ f^{(2)}=f'',\quad f^{(3)}=f''', \]

    y así sucesivamente, y\[ \frac{d^nf}{ dx^n}=f^{(n)}. \]

    Para derivar fórmulas de diferenciación para funciones elementales tales como\(\sin x\)\(\cos x\),, y\(e^x\) directamente de Definition~ requiere estimaciones basadas en las propiedades de estas funciones. Ya que esto se hace en cálculo, no vamos a repetirlo aquí.

    Si\(f(x)\) es la posición de una partícula en el tiempo\(x\ne x_0\), el cociente de diferencia\[ \frac{f(x)-f(x_0)}{ x-x_0} \] es la velocidad promedio de la partícula entre tiempos\(x_0\) y\(x\). A medida que\(x\) se aproxima\(x_0\), el promedio se aplica a intervalos cada vez más cortos. Por lo tanto, tiene sentido considerar el límite, si existe, como el {} de la partícula. Esta interpretación puede ser útil aunque no\(x\) sea el tiempo, por lo que a menudo consideramos\(f'(x_0)\) como el {}, independientemente de la naturaleza específica de la variable\(x\). La derivada también tiene una interpretación geométrica. La ecuación de la línea a través de dos puntos\((x_0,f(x_0))\) y\((x_1,f(x_1))\) en la curva\(y=f(x)\) (Figura~) es\[ y=f(x_0)+\frac{f(x_1)-f(x_0)}{ x_1-x_0} (x-x_0). \]

    Variante\(x_1\) genera líneas a través de\((x_0,f(x_0))\) que giran en la línea\[\begin{equation}\label{eq:2.3.2} y=f(x_0)+f'(x_0)(x-x_0) \end{equation}\]

    como\(x_1\) enfoques\(x_0\). Este es el {} a la curva\(y=f(x)\) en el punto\((x_0,f(x_0))\). Figura~ representa la situación para diversos valores de\(x_1\).

    6pt

    12pt

    Aquí hay una definición menos intuitiva de la línea tangente: Si la función\[ T(x)=f(x_0)+m(x-x_0) \] se aproxima\(f\) tan bien cerca\(x_0\) que\[ \lim_{x\to x_0}\frac{f(x)-T(x)}{ x-x_0}=0, \] decimos que la línea\(y=T(x)\) es {}.

    Esta línea tangente existe si y solo si\(f'(x_0)\) existe, en cuyo caso\(m\) se determina de manera única por\(m=f'(x_0)\) (Ejercicio~). Así, es la ecuación de la línea tangente.

    Utilizaremos el siguiente lema para estudiar funciones diferenciables.

    Definir\[\begin{equation} \label{eq:2.3.4} E(x)=\left\{\casespace\begin{array}{ll} \dst\frac{f(x)-f(x_0)}{ x-x_0}- f'(x_0),&x\in D_f\mbox{ and }x\ne x_0,\\[2\jot] 0,&x=x_0. \end{array}\right. \end{equation}\] Resolver para\(f(x)\) rendimientos si\(x\ne x_0\), y es obvio si\(x=x_0\). Definición~ implica eso\(\lim_{x\to x_0}E(x)=0\). Definimos\(E(x_0)=0\) hacer\(E\) continuo en\(x_0\).

    Dado que el lado derecho de es continuo en\(x_0\), también lo es el izquierdo. Esto arroja el siguiente teorema.

    Lo contrario de este teorema es falso, ya que una función puede ser continua en un punto sin ser diferenciable en el punto.

    El siguiente teorema debe ser familiar a partir del cálculo.

    La prueba se logra formando los cocientes de diferencia apropiados y aplicando Definición~ y Teorem~. Vamos a probar

    y dejarte el resto a ti (Ejercicios~, y).

    El truco es sumar y restar la cantidad correcta en el numerador del cociente de diferencia para\((fg)'(x_0)\); así,\[\begin{eqnarray*} \frac{f(x)g(x)-f(x_0)g(x_0)}{ x-x_0}\ar= \frac{f(x)g(x)-f(x_0)g(x)+f(x_0)g(x)-f(x_0)g(x_0)}{ x-x_0}\\ \ar=\frac{f(x)-f(x_0)}{ x-x_0} g(x)+f(x_0)\frac{g(x)-g(x_0)}{ x-x_0}. \end{eqnarray*}\] Los cocientes de diferencia en el enfoque correcto\(f'(x_0)\) y\(g'(x_0)\) como\(x\) enfoques\(x_0\), y\(\lim_{x\to x_0}g(x)=g(x_0)\) (Teorem~). Esto demuestra

    .

    Aquí está la regla para diferenciar una función compuesta.

    Ya que\(f\) es diferenciable en\(g(x_0)\), Lemma~ implica que\[ f(t)-f(g(x_0))=[f'(g(x_0))+E(t)][t-g(x_0)], \] donde\[\begin{equation}\label{eq:2.3.9} \lim_{t\to g(x_0)} E(t)=E(g(x_0))=0. \end{equation}\] Dejar\(t=g(x)\) rendimientos\[ f(g(x))-f(g(x_0))=[f'(g(x_0))+E(g(x))][g(x)-g(x_0)]. \] Desde\(h(x)=f(g(x))\), esto implica que Since\[\begin{equation}\label{eq:2.3.10} \frac{h(x)-h(x_0)}{ x-x_0}=[f'(g(x_0))+E(g(x))] \frac{g(x)-g(x_0)}{ x-x_0}. \end{equation}\]\(g\) es continuo en\(x_0\) (Teorem~), y Teorem~ implica que\[ \lim_{x\to x_0} E(g(x))=E(g(x_0))=0. \] Por lo tanto, implica que \[ h'(x_0)=\lim_{x\to x_0}\frac{h(x)-h(x_0)}{ x-x_0}=f'(g(x_0)) g'(x_0), \]como se ha dicho.

    Puede parecer razonable justificar la regla de la cadena escribiendo\[\begin{eqnarray*} \frac{h(x)-h(x_0)}{ x-x_0}\ar=\frac{f(g(x))-f(g(x_0))}{x-x_0}\\ \ar=\frac{f(g(x))-f(g(x_0))}{ g(x)-g(x_0)}\, \frac{g(x)-g(x_0)}{ x-x_0} \end{eqnarray*}\] y argumentando que\[ \lim_{x\to x_0} \frac{f(g(x))-f(g(x_0))}{ g(x)-g(x_0)}=f'(g(x_0)) \]

    (porque\(\lim_{x\to x_0} g(x)=g(x_0))\) y\[ \lim_{x\to x_0}\frac{g(x)-g(x_0)}{ x-x_0}=g'(x_0). \] Sin embargo, esta no es una prueba válida (Ejercicio~).

    Los límites unilaterales de los cocientes de diferencia como y en Ejemplo~ se llaman {} o {}. Es decir, si\(f\) se define en\([x_0,b)\), el {} se define para que sea\[ f_+'(x_0)=\lim_{x\to x_0+}\frac{f(x)-f(x_0)}{ x-x_0} \] si el límite existe, mientras que si\(f\) se define on\((a,x_0]\), el {} se define para que sea\[ f_-'(x_0)=\lim_{x\to x_0-}\frac{f(x)-f(x_0)}{ x-x_0} \] si el límite existe. Teorem~ implica que\(f\) es diferenciable en\(x_0\) si y solo si\(f_+'(x_0)\) y\(f_-'(x_0)\) existir y son iguales, en cuyo caso\[ f'(x_0)=f_+'(x_0)=f_-'(x_0). \]

    En Ejemplo~,\(f_+'(0)=1\) y\(f_-'(0)=-1\).

    Este ejemplo muestra que existe una diferencia entre una derivada unilateral y un límite unilateral de una derivada, ya que\(f_+'(0)=0\), pero, de,\(f'(0+)=\lim_{x\to 0+}f'(x)\) no existe. También muestra que un derivado puede existir en una vecindad de un punto\(x_0\) (\(=0\)en este caso), pero ser discontinuo en\(x_0\).

    Ejercicio~ justifica el método utilizado en Ejemplo~\(f'(x)\) para calcular\(x\ne0\).

    Decimos que\(f(x_0)\) es un {} de\(f\) si hay un\(\delta>0\) tal que\(f(x)-f(x_0)\) no cambia de signo en\[\begin{equation}\label{eq:2.3.13} (x_0-\delta,x_0+\delta)\cap D_f. \end{equation}\] Más específicamente,\(f(x_0)\) es un {} de\(f\) si\[\begin{equation}\label{eq:2.3.14} f(x) \le f(x_0) \end{equation}\] o un {} de\(f\) si\[\begin{equation}\label{eq:2.3.15} f(x)\ge f(x_0) \end{equation}\] para todos\(x\) en el conjunto. El punto\(x_0\) se llama un {} de\(f\), o, más específicamente, un {} o {} de\(f\).

    6pt

    12pt

    4pt

    Es geométricamente plausible que si la curva\(y=f(x)\) tiene una tangente en un punto extremo local de\(f\), entonces la tangente debe ser horizontal; es decir, tener pendiente cero. (Por ejemplo, en Figura~, ver\(x=1\),\(x=3\), y cada\(x\) en\((-1,-1/2)\).) El siguiente teorema muestra que esto debe ser así. 4pt

    4pt

    Demostraremos que no\(x_0\) es un punto extremo local de\(f\) si\(f'(x_0)\ne0\). De Lemma~,\[\begin{equation}\label{eq:2.3.16} \frac{f(x)-f(x_0)}{ x-x_0}=f'(x_0)+E(x), \end{equation}\] donde\(\lim_{x\to x_0} E(x)=0\). Por lo tanto\(f'(x_0)\ne0\), si, hay\(\delta>0\) tal que\[ |E(x)|<|f'(x_0)|\mbox{\quad if\quad} |x-x_0|<\delta, \] y el lado derecho de debe tener la misma señal que\(f'(x_0)\) para\(|x-x_0|<\delta\). Ya que lo mismo es cierto del lado izquierdo, se\(f(x)-f(x_0)\) debe cambiar de señal en cada barrio de\(x_0\) (ya que\(x-x_0\) hace). Por lo tanto, ni ni se puede sostener para todos\(x\) en ningún intervalo sobre\(x_0\). 4pt

    Si\(f'(x_0)=0\), decimos que\(x_0\) es un {} de\(f\). Teorem~ dice que cada punto extremo local de\(f\) en el que\(f\) es diferenciable es un punto crítico de\(f\). Lo contrario es falso. Por ejemplo,\(0\) es un punto crítico de\(f(x)=x^3\), pero no un punto extremo local. 4pt

    El uso del Teorem~ para encontrar puntos extremos locales está cubierto en el cálculo, por lo que no lo perseguiremos aquí. Sin embargo, usaremos Teorem~ para probar el siguiente teorema fundamental, que dice que si una curva\(y=f(x)\) se cruza con una línea horizontal en\(x=a\)\(x=b\) y y tiene una tangente at\((x,f(x))\) para cada\(x\)\((a,b)\) entrada, entonces hay un punto\(c\) en\((a,b)\) tal que la tangente a la curva en\((c,f(c))\) es horizontal (Figura~).

    6pt

    12pt

    Dado que\(f\) es continuo\([a,b]\),\(f\) alcanza un valor máximo y un valor mínimo on\([a,b]\) (Teorem~). Si estos dos valores extremos son los mismos, entonces\(f\) es constante on\((a,b)\), así\(f'(x)=0\) para todos\(x\) adentro\((a,b)\). Si los valores extremos difieren, entonces al menos uno debe ser alcanzado en algún punto\(c\) del intervalo abierto\((a,b)\), y\(f'(c)=0\), por Teorem~.

    Una derivada puede existir en un intervalo\([a,b]\) sin ser continua\([a,b]\). Sin embargo, un teorema de valor intermedio similar al Teorem~ se aplica a las derivadas.

    Supongamos primero eso\[\begin{equation}\label{eq:2.3.17} f'(a)<\mu<f'(b) \end{equation}\] y\[\begin{equation}\label{eq:2.3.18} g'(x)=f'(x)-\mu,\quad a\le x\le b, \end{equation}\] definamos\[ g(x)=f(x)-\mu x. \] Entonces e implica que\[\begin{equation}\label{eq:2.3.19} g'(a)<0\mbox{\quad and\quad} g'(b)>0. \end{equation}\] Since\(g\) es continuo en\([a,b]\),\(g\) alcanza un mínimo en algún momento\(c\) en\([a,b]\). Lemma~ e implica que hay\(\delta>0\) tal que\[ g(x)<g(a),\quad a<x<a+\delta,\mbox{\quad and\quad} g(x)<g(b),\quad b-\delta<x<b \]

    (Ejercicio~), y por lo tanto\(c\ne a\) y\(c\ne b\). De ahí,\(a<c<b\), y por lo tanto\(g'(c)=0\), por Teorem~. De,\(f'(c)=\mu\).

    El comprobante para el caso donde se\(f'(b)<\mu<f'(a)\) pueda obtener aplicando este resultado a\(-f\).

    La función\[ h(x)=[g(b)-g(a)]f(x)-[f(b)-f(a)]g(x) \] es continua\([a,b]\) y diferenciable en\((a,b)\), y\[ h(a)=h(b)=g(b)f(a)-f(b)g(a). \] Por lo tanto, el teorema de Rolle implica que\(h'(c)=0\) para algunos\(c\) en\((a,b)\). Ya que\[ h'(c)=[g(b)-g(a)]f'(c)-[f(b)-f(a)]g'(c), \] esto implica.

    El siguiente caso especial de Teorem~ es lo suficientemente importante como para ser expresado por separado.

    Aplicar Teorem~ con\(g(x)=x\).

    Teorem~ implica que la tangente a la curva\(y=f(x)\) en\((c,f(c))\) es paralela a la línea que conecta los puntos\((a,f(a))\) y\((b,f(b))\) sobre la curva (Figura ~, página).

    Si\(f\) es diferenciable en\((a,b)\) y\(x_1\),\(x_2\in (a,b)\) entonces\(f\) es continuo en el intervalo cerrado con puntos finales\(x_1\) y\(x_2\) y diferenciable en su interior. De ahí, el teorema del valor medio implica que\[ f(x_2)-f(x_1)=f'(c)(x_2-x_1) \] para algunos\(c\) entre\(x_1\) y\(x_2\). (Esto es cierto ya sea $x_1<x_2 $ o\(x_2<x_1\).) De esto se derivan los tres teoremas siguientes.

    Una función que satisface una desigualdad como para todos\(x\) y\(x'\) en un intervalo se dice que satisface un {} en el intervalo.

    6pt

    12pt

    El método del Teorem~ para encontrar límites de la suma, diferencia, producto y cociente de funciones se descompone en relación con formas indeterminadas. El teorema del valor medio generalizado (Teorem~) conduce a un método para evaluar límites de formas indeterminadas.

    Demostramos el teorema por finito\(L\) y dejamos el caso donde\(L=\pm\infty\) a ti (Ejercicio~).

    Supongamos que\(\epsilon >0\). De, hay un\(x_0\) en\((a,b)\) tal que\[\begin{equation}\label{eq:2.4.5} \left|\frac{f'(c)}{g'(c)}-L\right|<\epsilon\mbox{\quad if\quad} x_0 <c<b. \end{equation}\] Teorem~ implica que si\(x\) y\(t\) están en\([x_0, b)\), entonces hay un\(c\) entre ellos, y por lo tanto en\((x_0,b)\), tal que\[\begin{equation}\label{eq:2.4.6} [g(x)-g(t)]f'(c)=[f(x)-f(t)]g'(c). \end{equation}\] Desde que no\(g'\) tiene ceros en\((a,b)\), Teorem~ implica que \[ g(x)-g(t)\ne0\mbox{\quad if\quad} x, t\in (a,b). \]Esto quiere decir que\(g\) no puede tener más de un cero adentro\((a,b)\). Por lo tanto, podemos elegir\(x_0\) para que, además de, no\(g\) tenga ceros en\([x_0,b)\). Entonces se puede reescribir como\[ \frac{f(x)-f(t)}{ g(x)-g(t)}=\frac{f'(c)}{ g'(c)}, \] así implica que\[\begin{equation}\label{eq:2.4.7} \left|\frac{f(x)-f(t)}{ g(x)-g(t)}-L\right|<\epsilon \mbox{\quad if\quad} x, t\in [x_0,b). \end{equation}\]

    Si se mantiene, deje que\(x\) se fije en\([x_0,b)\), y considere la función\[ G(t)=\frac{f(x)-f(t)}{ g(x)-g(t)}-L. \] From,\[ \lim_{t\to b-}f(t)=\lim_{t\to b-}g(t)=0, \] entonces\[\begin{equation}\label{eq:2.4.8} \lim_{t\to b-}G(t)=\frac{f(x)}{g(x)}-L. \end{equation}\]

    Ya que\[ |G(t)|<\epsilon\mbox{\quad if\quad} x_0<t<b, \] a causa de, implica que\[ \left|\frac{f(x)}{ g(x)}-L\right|\le\epsilon. \] Esto sostiene para todos\(x\) en\((x_0,b)\), lo que implica.

    La prueba bajo suposición es más complicada. De nuevo elige\(x_0\) para que se mantenga y no\(g\) tenga ceros en\([x_0,b)\). Dejando\(t=x_0\) entrar, vemos que\[\begin{equation} \label{eq:2.4.9} \left|\frac{f(x)-f(x_0)}{ g(x)-g(x_0)}-L\right|<\epsilon \mbox{\quad if\quad} x_0\le x<b. \end{equation}\] Desde\(\lim_{x\to b-}f(x)=\pm\infty\), podemos elegir\(x_1>x_0\) para que\(f(x)\ne0\) y\(f(x)\ne f(x_0)\) si\(x_1<x<b\). Entonces\[ u(x)=\frac{1-g(x_0)/g(x)}{1-f(x_0)/f(x)} \] se define la función y no cero si\(x_1<x<b\), y\[\begin{equation}\label{eq:2.4.10} \lim_{x\to b-} u(x)=1, \end{equation}\] debido a.

    Ya que\[ \frac{f(x)-f(x_0)}{ g(x)-g(x_0)}=\frac{f(x)}{ g(x)}\ \frac{1-f(x_0)/f(x)}{ 1-g(x_0)/g(x)}=\frac{f(x)}{ g(x)u(x)}, \] implica\[ \left|\frac{f(x)}{ g(x) u(x)}-L\right|<\epsilon\mbox{\quad if\quad} x_1<x<b, \] lo que se puede reescribir como\[\begin{equation} \label{eq:2.4.11} \left|\frac{f(x)}{ g(x)}-Lu(x)\right|<\epsilon |u(x)|\mbox{\quad if \quad}x_1<x<b. \end{equation}\] A partir de esto y el triángulo de la desigualdad,\[\begin{equation}\label{eq:2.4.12} \left|\frac{f(x)}{ g(x)}-L\right|\le\left|\frac{f(x)}{ g(x)}-Lu(x)\right| +|Lu(x)-L|\le\epsilon|u(x)|+|L|\,|u(x)-1|. \end{equation}\] Debido a, hay un punto\(x_2\) en\((x_1,b)\) tal que\[ |u(x)-1|<\epsilon\mbox{\quad and therefore \quad} |u(x)|<1+\epsilon \mbox{\quad if\quad} x_2<x<b. \] Esto,, e implica que\[ \left|\frac{f(x)}{ g(x)}-L\right|<\epsilon (1+\epsilon)+|L|\epsilon \mbox{\quad if\quad} x_2<x<b, \]

    lo que prueba bajo suposición.

    Teorem~ y la prueba dada aquí siguen siendo válidos si\(b=\infty\) y $x\ a b-$” se sustituye por\(x\to\infty\)” en todo momento. Solo se requieren cambios menores en la prueba para demostrar que teoremas similares son válidos para límites de la derecha, límites a\(-\infty\), y límites ordinarios (de dos lados). Los tomaremos como se nos ha dado.

    -.4em Decimos que\(f/g\) {}\(x\to b-\) if\[ \lim_{x\to b-}f(x)=\lim_{x\to b-}g(x)=0, \] o {} if\[ \lim_{x\to b-} f(x)=\pm\infty \] y\[ \lim_{x\to b-} g(x)=\pm\infty. \] Las definiciones correspondientes para\(x\to b+\) y\(x\to\pm\infty\) son similares. Si\(f/g\) es de una de estas formas como\(x\to b-\) y como\(x\to b+\), entonces decimos que es de esa forma como\(x\to b\).

    Al usar la regla de L'Hospital solemos escribir, por ejemplo,\[\begin{equation}\label{eq:2.4.13} \lim_{x\to b}\frac{f(x)}{ g(x)}=\lim_{x\to b}\frac{f'(x)}{ g'(x)} \end{equation}\] para luego tratar de encontrar el límite a la derecha. Esto es conveniente, pero técnicamente incorrecto, ya que es cierto solo si el límite a la derecha existe en los reales extendidos. Puede suceder que el límite de la izquierda exista pero el de la derecha no. En este caso, es incorrecto.

    Decimos que un producto\(fg\) {} si uno de los factores se acerca\(0\) y el otro se acerca\(\pm\infty\) como\(x\to b-\). En este caso, puede ser útil aplicar la regla de L'Hospital después de escribir\[ f(x)g(x)=\frac{f(x)}{1/g(x)}\mbox{\quad or\quad} f(x)g(x)=\frac{g(x)}{1/f(x)}, \] ya que una de estas proporciones es de la forma\(0/0\) y la otra es de la forma\(\infty/\infty\) como\(x\to b-\).

    Declaraciones similares se aplican a los límites como\(x\to b+\)\(x\to b\),, y\(x\to \pm\infty\).

        \enlargethispage{100pt}

    Una diferencia\(f-g\) {} si\[ \lim_{x\to b-} f(x)=\lim_{x\to b-} g(x)=\pm\infty. \] En este caso, puede ser posible manipular\(f-g\) en una expresión que ya no es indeterminada, o es de la forma\(0/0\) o\(\infty/\infty\) como\(x\to b-\). Observaciones similares se aplican a los límites como\(x\to b+\),\(x\to b\), o\(x\to\pm\infty\).

    La función\(f^g\) está definida por\[ f(x)^{g(x)}=e^{g(x)\log f(x)}=\exp(g(x)\log f(x)) \] para todos\(x\) tales que\(f(x)>0\). Por lo tanto, si\(f\) y\(g\) están definidos y\(f(x)>0\) en un intervalo\((a,b)\), Ejercise~ implica que\[\begin{equation}\label{eq:2.4.14} \lim_{x\to b-}[f(x)]^{g(x)}=\exp\left(\lim_{x\to b-} g(x)\log f(x)\right) \end{equation}\] si\(\lim_{x\to b-}g(x)\log f(x)\) existe en los reales extendidos. (Si este límite es\(\pm\infty\) entonces es válido si definimos\(e^{-\infty}=0\) y\(e^\infty=\infty\).) El producto\(g\log f\) puede ser de la forma\(0\cdot\infty\) de tres maneras como\(x\to b-\): 10pt

    10pt En estos tres casos, decimos que\(f^g\) {}. Definiciones similares se aplican a los límites como\(x\to b+\)\(x\to b\),, y\(x\to\pm\infty\).

    10pt

    10pt

    \ begin {exerciselist}

    Demostrar Teoremo~ para el caso donde\(\lim_{x\to b-}f'(x)/g'(x)=\pm\infty\).

    \(\dst\lim_{x\to 0}\frac{\tan^{-1}x}{\sin^{-1}x}\)&\(\dst\lim_{x\to 0}\frac{1-\cos x}{\log(1+x^2)}\) &

    \(\dst\lim_{x\to 0+}\frac{1+\cos x}{e^x-1}\)\\ final {tabular}

    \(\dst\lim_{x\to\pi}\frac{\sin nx}{\sin x}\)&\(\dst\lim_{x\to 0}\frac{\log(1+x)}{ x}\) &

    \(\dst\lim_{x\to\infty}e^x\sin e^{-x^2}\)\\ final {tabular}

    \(\dst\lim_{x\to\infty} x\sin(1/x)\)&\(\dst\lim_{x\to\infty}\sqrt{x}(e^{-1/x}-1)\) &

    \(\dst\lim_{x\to 0+}\tan x\log x\)\ end {tabular}

    \(\dst\lim_{x\to\pi}\sin x\log (|\tan x|)\)&

    \(\dst\lim_{x\to 0+} {\left[\frac{1}{x}+\log(\tan x)\right]}\)\\ final {tabular}

    \(\dst\lim_{x\to\infty} (\sqrt{x+1}-\sqrt{x})\)&

    \(\dst\lim_{x\to 0} \left(\frac{1}{ e^x-1}-\frac{1}{ x}\right)\)\\ final {tabular}

    \(\dst\lim_{x\to 0} (\cot x-\csc x)\)&

    \(\dst\lim_{x\to 0} {\left(\frac{1}{\sin x}-\frac{1}{ x}\right)}\)\\ final {tabular}

    \(\dst\lim_{x\to\pi} |\sin x|^{\tan x}\)&

    \(\dst\lim_{x\to\pi/2} |\tan x|^{\cos x}\)\\ final {tabular}

    \(\dst\lim_{x\to 0} |\sin x|^x\)&

    \(\dst\lim_{x\to 0}(1+x)^{1/x}\)\\ final {tabular}

    \(\dst\lim_{x\to\infty} x^{\sin(1/x)}\)&

    \(\dst\lim_{x\to 0} {\left(\frac{x}{1-\cos x}-\frac{2}{ x}\right)}\)\\ final {tabular}

    \(\dst\lim_{x\to 0+} x^\alpha\log x\)&

    \(\dst\lim_{x\to e} \frac{\log(\log x)}{\sin(x-e)}\)\\ final {tabular}

    \(\dst\lim_{x\to\infty}{\left(\frac{x+1}{ x-1}\right)}^{\sqrt{x^2-1}}\)&

    \(\dst\lim_{x\to 1+}{\left( \frac{x+1}{ x-1}\right)}^{\sqrt{x^2-1}}\)\\ final {tabular}

    \(\dst\lim_{x\to\infty}\frac{(\log x)^\beta}{ x}\)&

    \(\dst\lim_{x\to\infty}(\cosh x-\sinh x)\)\\ final {tabular}

    \(\dst\lim_{x\to\infty}(x^\alpha-\log x)\)&

    \(\dst\lim_{x\to-\infty}e^{x^2}\sin(e^x)\)\\ final {tabular}

    \(\dst\lim_{x\to\infty} x(x+1)\left[\log(1+1/x)\right]^2\)&

    \(\dst\lim_{x\to 0}\frac{\sin x-x+x^3/6}{x^5}\)\\ final {tabular}

    \(\dst\lim_{x\to\infty}\frac{e^x}{ x^\alpha}\)&

    \(\dst\lim_{x\to 3\pi/2-} e^{\tan x}\cos x\)\\ final {tabular}

    \(\dst\lim_{x\to1+}\dst{(\log x)}^\alpha\log(\log x)\)&

    \(\dst\lim_{x\to\infty}\frac{x^x}{ x\log x}\)\ end {tabular}

    \(\dst\lim_{x\to\pi/2}(\sin x)^{\tan x}\)

    \(\dst\lim_{x\to 0} \frac{e^x-\dst\sum_{r=0}^n{x^r}{ r!}}{ x^n}\quad (n=\mbox{ integer}\ge1)\)

    \(\dst\lim_{x\to 0} \frac{\sin x-\dst{\sum_{r=0}^n (-1)^r\frac{x^{2r+1} }{(2r+1)!}}}{ x^{2n+1}}\quad (n=\mbox{ integer}\ge0)\)

    \(\dst{\lim_{x\to 0}\frac{e^{-1/x^2}}{ x^n}=0}\)(\(n=\)entero)

    Los {} están definidos por\(L_0(x)=x\) y\[ L_n(x)=\log(L_{n-1}(x)),\quad x>a_n,\quad n\ge1, \] dónde\(a_1=0\) y\(a_n=e^{a_{n-1}}, n\ge1\). Demostrar que

    \(f\)Sea positivo y diferenciable\((0,\infty)\), y supongamos que\[ \lim_{x\to\infty}\frac{f'(x)}{ f(x)}=L,\mbox{\quad where \quad} 0<L\le\infty. \] Definir\(f_0(x)=x\) y\[ f_n(x)=f\left(f_{n-1}(x)\right),\quad n\ge1. \] Usar la regla de L'Hospital para demostrar que\[ \lim_{x\to\infty}\frac{(f_n(x))^\alpha}{ f_{n-1}(x)}=\infty \mbox{\quad if \quad}\alpha>0\mbox{\quad and \quad} n\ge1. \]

    Dejar\(f\) ser diferenciable en algún barrio eliminado\(N\) de\(x_0\), y supongamos que\(f\) y no\(f'\) tienen ceros en\(N\). Encuentra

    Supongamos que\(f\) y\(g\) son diferenciables y no\(g'\) tiene ceros encendidos\((a,b)\). Supongamos también eso\(\lim_{x\to b-}f'(x)/g'(x)=L\) y ya sea\[ \lim_{x\to b-}f(x)=\lim_{x\to b-}g(x)=0 \] o\[ \lim_{x\to b-}f(x)=\infty\mbox{\quad and \quad}\lim_{x\to b-}g(x)=\pm\infty. \] Find\(\lim_{x\to b-}(1+f(x))^{1/g(x)}\).

    Distinguimos entre\(\infty\cdot\infty\)\((=\infty)\) y\((-\infty)\infty\)\((=-\infty)\) y entre\(\infty+\infty\)\((=\infty)\) y\(-\infty-\infty\)\((=-\infty)\). ¿Por qué no distinguimos entre\(0\cdot\infty\) y\(0\cdot (-\infty)\),\(\infty-\infty\) y\(-\infty+\infty\),\(\infty/\infty\) y\(-\infty/\infty\), y y\(1^\infty\) y\(1^{-\infty}\)?

    \ end {lista de ejercicios}

    A {} es una función de la forma\[\begin{equation}\label{eq:2.5.1} p(x)=a_0+a_1(x-x_0)+\cdots+a_n(x-x_0)^n, \end{equation}\] donde\(a_0\),,\(a_n\) y\(x_0\) son constantes. Dado que es fácil calcular los valores de un polinomio, se ha dedicado un esfuerzo considerable a utilizarlos para aproximar funciones más complicadas. El teorema de Taylor es uno de los resultados más antiguos e importantes sobre esta cuestión.

    Se dice que el polinomio está escrito {}\(x-x_0\), y es {}\(n\) si\(a_n\ne0\). Si queremos dejar abierta la posibilidad de eso\(a_n=0\), decimos que\(p\) es de grado\(\le n\). En particular, un polinomio constante\(p(x)=a_0\) es de grado cero si\(a_0\ne0\). Si\(a_0=0\), para que eso se\(p\) desvanezca de manera idéntica, entonces no\(p\) tiene grado según nuestra definición, lo que requiere que al menos un coeficiente sea distinto de cero. Por conveniencia decimos que el polinomio idéntico cero\(p\) tiene grado\(-\infty\). (Cualquier número negativo haría tan bien como\(-\infty\). El punto es que con esta convención, la afirmación de que\(p\) es un polinomio de grado\(\le n\) incluye la posibilidad de que\(p\) sea idénticamente cero.)

    Vimos en Lemma~ que si\(f\) es diferenciable en\(x_0\), entonces\[ f(x)=f(x_0)+f'(x_0)(x-x_0)+E(x)(x-x_0), \]

    donde\[ \lim_{x\to x_0} E(x)=0. \] Para generalizar este resultado, primero lo reafirmamos: el polinomio\[ T_1(x)=f(x_0)+f'(x_0)(x-x_0), \] que es de grado\(\le1\) y satisface se\[ T_1(x_0)=f(x_0),\quad T'_1(x_0)=f'(x_0), \] aproxima\(f\) tan bien cerca de\(x_0\) eso\[\begin{equation} \label{eq:2.5.2} \lim_{x\to x_0}\frac{f(x)-T_1(x)}{ x-x_0}=0. \end{equation}\]

    Ahora supongamos que\(f\) tiene\(n\) derivados en\(x_0\) y\(T_n\) es el polinomio de grado~\(\le~n\) tal que\[\begin{equation}\label{eq:2.5.3} T^{(r)}_n (x_0)=f^{(r)}(x_0),\quad 0\le r\le n. \end{equation}\] ¿Qué tan bien se\(T_n\)\(f\) aproxima\(x_0\)?

    Para responder a esta pregunta, primero debemos encontrar\(T_n\). Ya que\(T_n\) es un polinomio de grado~\(\le~n\), se puede escribir como\[\begin{equation}\label{eq:2.5.4} T_n(x)=a_0+a_1(x-x_0)+\cdots+a_n(x-x_0)^n, \end{equation}\] donde\(a_0\),,\(a_n\) son constantes. Diferenciar rendimientos\[ T^{(r)}_n(x_0)=r!a_r,\quad 0\le r\le n, \] así determina de\(a_r\) manera única como\[ a_r=\frac{f^{(r)}(x_0)}{ r!},\quad 0\le r\le n. \] Por lo tanto,\[\begin{eqnarray*} T_n(x)\ar=f(x_0)+\frac{f'(x_0)}{1!} (x-x_0)+\cdots+\frac{f^{(n)}(x_0) }{ n!} (x-x_0)^n\\ \ar=\sum_{r=0}^n\frac{f^{(r)} (x_0)}{ r!} (x-x_0)^r. \end{eqnarray*}\] llamamos\(T_n\) {} {}.

    El siguiente teorema describe cómo\(T_n\) se aproxima\(f\) cerca\(x_0\).

    La prueba es por inducción. \(P_n\)Sea la aseveración del teorema. De nosotros sabemos que es cierto si\(n=1\); es decir,\(P_1\) es verdad. Ahora supongamos que eso\(P_n\) es cierto para algún entero\(n\ge1\), y\(f^{(n+1)}\) existe. Dado que la relación\[ \frac{f(x)-T_{n+1}(x)}{(x-x_0)^{n+1}} \] es indeterminada de la forma\(0/0\) como\(x\to x_0\), la regla de L'Hospital implica que\[\begin{equation}\label{eq:2.5.6} \lim_{x\to x_0}\frac{f(x)-T_{n+1}(x)}{(x-x_0)^{n+1}}=\frac{1}{ n+1}\lim_{x\to x_0}\frac{f'(x)-T'_{n+1}(x)}{(x-x_0)^n} \end{equation}\] si existe el límite sobre el derecho. Pero\(f'\) tiene un\(n\) th derivado en\(x_0\), y\[ T'_{n+1}(x)=\sum_{r=0}^n\frac{f^{(r+1)}(x_0)}{ r!}(x-x_0)^r \] es el polinomio\(n\) th Taylor de\(f'\) aproximadamente\(x_0\). Por lo tanto, el supuesto de inducción\(f'\), aplicado a, implica que\[ \lim_{x\to x_0}\frac{f'(x)-T'_{n+1}(x)}{(x-x_0)^n}=0. \] Esto e implica\[ \lim_{x\to x_0}\frac{f(x)-T_{n+1}(x)}{(x-x_0)^{n+1}}=0, \] lo que completa la inducción.

    Se puede mostrar (Ejercicio~) que si\[ p_n=a_0+a_1(x-x_0)+\cdots+a_n(x-x_0)^n \] es un polinomio de grado\(\le n\) tal que\[ \lim_{x\to x_0}\frac{f(x)-p_n(x)}{(x-x_0)^n}=0, \] entonces\[ a_r=\frac{f^{(r)}(x_0)}{ r!}; \] eso es,\(p_n=T_n\). Así,\(T_n\) es el único polinomio de grado\(\le n\) que se aproxima\(f\) cerca de\(x_0\) la manera indicada en.

    Teorem~ se puede replantear como una generalización de Lemma~.

    Definir\[ E_n(x)= \left\{\casespace\begin{array}{ll} \dst\frac{f(x)-T_n(x)}{(x-x_0)^n},&x\in D_f-\{x_0\},\\ 0,&x=x_0.\end{array}\right. \] Entonces implica eso\(\lim_{x\to x_0}E_n(x)=E_n(x_0)=0\), y es sencillo de verificar.

    Si\(q\) es un entero no negativo, entonces\(\dst\binom{q}{n}\) es el coeficiente binomial definido en Ejercicio~. En este caso, vemos a partir de ese\[ T_n(x)=(1+x)^q=f(x),\quad n\ge q. \]

    Lemma~ produce el siguiente teorema.

    Ya que\(f^{(r)}(x_0)=0\) para\(1\le r\le n-1\), implica que\[\begin{equation}\label{eq:2.5.10} f(x)-f(x_0)=\left[\frac{f^{(n)}(x_0)}{ n!}+E_n(x)\right] (x-x_0)^n \end{equation}\] en algún intervalo que contenga\(x_0\). Desde\(\lim_{x\to x_0} E_n(x)=0\) y\(f^{(n)}(x_0)\ne0\), hay\(\delta>0\) tal que\[ |E_n(x)|<\left|\frac{f^{(n)}(x_0)}{ n!}\right|\mbox{\quad if\quad} |x-x_0| <\delta. \]

    Esto e implica que\[\begin{equation}\label{eq:2.5.11} \frac{f(x)-f(x_0)}{(x-x_0)^n} \end{equation}\] tiene el mismo signo que\(f^{(n)}(x_0)\) si\(0<|x-x_0|<\delta\). Si\(n\) es impar el denominador de los cambios firmar en cada barrio de\(x_0\), y por lo tanto también debe hacerlo el numerador (ya que la relación tiene signo constante para\(0<|x-x_0|<\delta\)). En consecuencia,\(f(x_0)\) no puede ser un valor extremo local de\(f\). Esto prueba. Si\(n\) es par, el denominador de es positivo para\(x\ne x_0\), por lo que\(f(x)-f(x_0)\) debe tener el mismo signo que\(f^{(n)}(x_0)\) para\(0<|x-x_0|<\delta\). Esto demuestra

    .

    Para\(n=2\),

    se llama el {} para los puntos extremos locales.

    Teorem~ implica que el error en la aproximación\(f(x)\) por\(T_n(x)\) se aproxima a cero más rápido que\((x-x_0)^n\) como\(x\) enfoques\(x_0\); sin embargo, no da estimación del error al aproximar\(f(x)\) por\(T_n(x)\) para un {}\(x\). Por ejemplo, no proporciona ninguna estimación del error en la aproximación\[\begin{equation}\label{eq:2.5.12} e^{0.1}\approx T_2(0.1)=1+\frac{0.1}{1!}+\frac{(0.1)^2}{2!}=1.105 \end{equation}\] obtenida por ajuste\(n=2\) y\(x=0.1\) en. El siguiente teorema proporciona una manera de estimar errores de este tipo bajo la suposición adicional que\(f^{(n+1)}\) existe en un barrio de\(x_0\).

    Este teorema se desprende de una extensión del teorema del valor medio que probaremos a continuación. Por ahora, supongamos que Teorem~ es correcto, y aplicarlo.

    Consideramos ahora el teorema del valor medio extendido, que implica Teorem~ (Ejercicio~). En el siguiente teorema,\(a\) y\(b\) son los puntos finales de un intervalo, pero no asumimos que\(a<b\).

    La prueba es por inducción. El teorema del valor medio (Teorem~) implica la conclusión para\(n=0\). Ahora supongamos eso\(n\ge1\), y supongamos que la aseveración del teorema es verdadera con\(n\) reemplazada por\(n-1\). El lado izquierdo de se puede escribir como\[\begin{equation}\label{eq:2.5.18} f(b)-\sum_{r=0}^n\frac{f^{(r)}(a)}{ r!}(b-a)^r=K\frac{(b-a)^{n+1}}{(n+1)!} \end{equation}\] para algún número\(K\). Debemos probarlo\(K=f^{(n+1)}(c)\) para algunos\(c\) en\(I^0\). Para ello, consideremos la función auxiliar\[ h(x)=f(x)-\sum_{r=0}^n\frac{f^{(r)}(a)}{ r!}(x-a)^r-K\frac{(x-a)^{n+1}}{ (n+1)!}, \] que satisface\[ h(a)=0,\quad h(b)=0, \] (esta última a causa de) y es continua en el intervalo cerrado\(I\) y diferenciable en\(I^0\), con\[\begin{equation}\label{eq:2.5.19} h'(x)=f'(x)-\sum_{r=0}^{n-1}\frac{f^{(r+1)}(a)}{ r!}(x-a)^r-K\frac{(x-a)^n}{n!}. \end{equation}\] Por lo tanto, el teorema de Rolle (Teorem~) implica que\(h'(b_1)=0\) para algunos\(b_1\) en \(I^0\); así, de,\[ f'(b_1)-\sum_{r=0}^{n-1}\frac{f^{(r+1)}(a)}{ r!}(b_1-a)^r-K\frac{(b_1-a)^n}{n!}=0. \] Si escribimos temporalmente\(f'=g\), esto se convierte en\[\begin{equation}\label{eq:2.5.20} g(b_1)-\sum_{r=0}^{n-1}\frac{g^{(r)}(a)}{ r~}(b_1-a)^r-K\frac{(b_1-a)^n}{n!}=0. \end{equation}\]

    Ya que\(b_1\in I^0\), las hipótesis sobre\(f\) implican que\(g\) es continuo en el intervalo cerrado\(J\) con puntos finales\(a\) y\(b_1\),\(g^{(n)}\) existe en\(J^0\), y, si\(n\ge1\),\(g'\),,\(g^{(n-1)}\) existen y son continuos en\(a\) (también en\(b_1\), pero esto no es importante). La hipótesis de inducción, aplicada\(g\) en el intervalo\(J\), implica que\[ g(b_1)-\sum_{r=0}^{n-1}\frac{g^{(r)}(a)}{ r!} (b_1-a)^r=\frac{g^{(n)}(c)}{n!}(b_1-a)^n \] para algunos\(c\) en\(J^0\). Comparando esto y recordando que\(g=f'\) rinde\[ K=g^{(n)}(c)=f^{(n+1)}(c). \] Desde que\(c\) está en\(I^0\), esto completa la inducción.


    This page titled 2.1: Funciones y límites is shared under a CC BY-NC-SA 3.0 license and was authored, remixed, and/or curated by William F. Trench.